Se encontraron 302 coincidencias

por ésta
Sab 01 Oct, 2022 4:07 pm
Foro: Problemas Archivados de Teoría de Números
Tema: Iberoamericana 2022 - Problema 2
Respuestas: 2
Vistas: 611

Re: Iberoamericana 2022 - Problema 2

Veamos que Ana tiene estrategia ganadora. Sean $s_1 \leq s_2 \leq \ldots \leq s_n \leq \ldots$ los números de $S$ ordenados de menor a mayor. Numeramos las casillas del tablero de $0$ a $2021$ de derecha a izquierda de modo que Ana juega en las casillas impares. La idea es que cuando a Ana le toque...
por ésta
Lun 23 May, 2022 5:23 am
Foro: Problemas Archivados de Combinatoria
Tema: APMO 2022 P4
Respuestas: 3
Vistas: 348

Re: APMO 2022 P4

Cuidado Ignorando la última caja y las bolitas $2^{k−1}+2≤j≤2^k+1$, tuvimos una situación en la cual había $2^{k−1}+1$ bolitas y $k$ cajas, y nos quedó solita la bolita $2^{k−1}+1$ en la primer caja, de donde $(2^{k−1}+1,k)$ resultaría ganador contradiciendo la elección de $k+1$ como el menor con es...
por ésta
Sab 14 May, 2022 3:09 pm
Foro: Problemas Archivados de Combinatoria
Tema: Selectivo IMO 2022 P5
Respuestas: 3
Vistas: 832

Re: Selectivo IMO 2022 P5

Paso 1: En un ordenamiento válido no hay dos números pares consecutivos. Demo: Si tenemos $a,b,c$ consecutivos con $b$ par, $b\mid a+c\Rightarrow 2\mid a+c$ por lo que $a$ y $c$ tienen la misma paridad. Por lo que si $b$ es adyacente a un par, sus dos vecinos son pares. Esto implica que si hay dos ...
por ésta
Lun 18 Abr, 2022 6:41 am
Foro: Problemas Archivados de Combinatoria
Tema: Selectivo Cono 2022 P2
Respuestas: 2
Vistas: 600

Re: Selectivo Cono 2022 P2

Cota: Supongamos que exista una coloración válida para $n$ y $k$. Es claro que $n \geq k$ y como $k > 1$ tenemos $n \geq 2$. Luego hay una segunda fila en el tablero y hay una columna que tiene una casilla pintada en la segunda fila. Miramos esa columna y una adyacente (siempre existe al menos una)....
por ésta
Jue 25 Mar, 2021 2:34 pm
Foro: Problemas Archivados de Teoría de Números
Tema: XX Rioplatense N3 P6
Respuestas: 5
Vistas: 1985

Re: XX Rioplatense N3 P6

Ahi lo arregle, había un menor en lugar de un menor igual. Fijate que el Lema 1 vale para $M+1$ inclusive y eso me permite usar los primos hasta $p_{M+1}$ inclusive para el Lema 2.
por ésta
Vie 22 Nov, 2019 8:44 pm
Foro: Problemas Archivados de Teoría de Números
Tema: APMO 2019 Problema 1
Respuestas: 4
Vistas: 3021

Re: APMO 2019 Problema 1

La solución de arriba tiene un problema: No es correcto como descarta el caso $\frac{b}{x}$ no entero, para poder decir cuando $a \mid bc$ con $a$ y $b$ coprimos entonces $a \mid c$ el número $c$ tiene que ser entero. Por ejemplo $5 \mid 10 = 6 \times \frac{5}{3}$ y no puedo decir que entonces $5 \m...
por ésta
Dom 11 Ago, 2019 7:55 pm
Foro: Problemas Archivados de Álgebra
Tema: Selectivo Ibero 2019 - Problema 2
Respuestas: 4
Vistas: 2135

Re: Selectivo Ibero 2019 - Problema 2

$S=x_2(x_2−x_0)+x_3(x_3−x_1)+x_4(x_4−x_2)+…+x_{2019}(x_{2019}−x_{2017})$ Lo podemos escribir como $S=x_2(x_1−x_0) + x_2(x_2-x_1)+x_3(x_2−x_1)+x_3(x_3-x_2)+x_4(x_3−x_2)+x_4(x_4−x_3) +\ldots + x_{2019}(x_{2018}−x_{2017})+x_{2019}(x_{2019}−x_{2018})$ Como la sucesión es creciente, ésta suma es por lo ...
por ésta
Mar 10 Jul, 2018 6:40 pm
Foro: Problemas Archivados de Combinatoria
Tema: IMO 2018 - P4
Respuestas: 3
Vistas: 2366

Re: IMO 2018 - P4

Veamos que el mayor $K$ posible es $100$. Para eso tenemos que ver dos cosas, que Ana puede asegurarse colocar $100$ piedras y que Beto puede asegurarse de que Ana no coloque más de $100$ piedras. Veamos primero que Ana puede asegurarse colocar $100$ piedras. Pintamos los lugares de blanco y negro ...
por ésta
Sab 27 Ene, 2018 12:05 am
Foro: Problemas Archivados de Teoría de Números
Tema: OFO 2018 Problema 1
Respuestas: 3
Vistas: 2701

OFO 2018 Problema 1

Determinar el menor entero positivo $N$ que cumple la siguiente propiedad: "Es imposible dividir los números $1,2,3,\ldots,2N$ en $N$ parejas de manera que las sumas de los números en cada pareja sean $N$ números primos distintos." (Para el valor hallado, justificar por qué es imposible fo...
por ésta
Vie 08 Dic, 2017 6:41 pm
Foro: Problemas Archivados de Teoría de Números
Tema: Rioplatense 2017 - N3 P6
Respuestas: 0
Vistas: 1496

Rioplatense 2017 - N3 P6

Para un entero fijo $n\in \mathbb{N}$, $n\geq 4$, y $l\in \mathbb{Z}$ sea $(l)_n\in [1,n]$ el menor resto positivo de $l$ módulo $n$. Dos sucesiones $a_1,\ldots ,a_k$ y $b_1,\ldots ,b_k$ con términos en $[1,n]$ son equivalentes si existe $t\in \mathbb{Z}$, con $t$ y $n$ coprimos, tal que la sucesión...